PT88.S1.Q17 (G3) - flowers game

matthiessemmatthiessem Alum Member
edited October 2019 in Logic Games 34 karma

Hey guys, could someone help me understand why (e) is the correct answer? I seem to be able to produce at least one possible world in which everyone has flower arrangements with at least one use of lilies in it (U gets the LL block), and in which there are two people with the HR block (U and Z). As far as I can tell, I've got the antecedent setup/questions mostly right. It's just this last question that flummoxes me.

Thank you so much! Sorry for violating any norms here: I just signed up for the course and haven't yet got a sense of the norms yet!

Admin. note: slightly edited discussion title to fit formatting guidelines: "PT#.S#.Q# (G#) - brief description"

Comments

  • youbbyunyoubbyun Alum Member
    edited October 2019 1755 karma

    For Z, you need 3 types of flowers. You have exactly 1h, 1r, what’s the other flower? If it’s L, you must have 2L’s, because you need 4 flowers per group. But wait, your 2L’s are already in U. And there can be only exactly 1 2L’s. And it’s already used up in U. So Z is the group with no L. It can have 2 G’s instead. Make sense?

    Thus we see that at least one arrangement contains no L.

  • matthiessemmatthiessem Alum Member
    34 karma

    Got it! Thank you so much. I'd accidentally left the Z group as closed out by "G/L" even after I placed the LL block with U.

    Good grief: I drew two more gameboards, one with 3 HR in total (T, U, and Z) and another with 4 HR in total (T, U, W, Z) and could reveal the "at least one group has no Ls" for those, but not the 2 HR total gameboard. So much time expended ;_;

  • Julie_NYCJulie_NYC Alum Member
    edited October 2019 24 karma

    Thanks!

  • 2ndTimestheCharm2ndTimestheCharm Alum Member
    edited October 2019 1810 karma

    I did this game for the first time today, and it may be wrong, but in my view the LL block can't be in U, it has to either be in T or in Z the whole game. My reasoning is that since U has exactly one R and at least one H, the H must double (since Z is the only one that can have just one R and one H), leaving U with R, H, H and either G or L. So the only place the LL block can go is T or Z.

    After subbing the rule for 17, we realize it has to be true that either T, U or Z won't have any Ls. 17 is certainly one to skip under time. I'm not going to spend too much time trying to foolproof this game, because what are the chances they'll do this same thing again next week? They'll do something else crazy hard, but it won't be this exactly.

Sign In or Register to comment.